Green's Theorem and simple closed curve

Click For Summary
The discussion focuses on using Green's Theorem to find the area enclosed by the curve defined by x^(2/3) + y^(2/3) = 4. The area can be calculated using the formula A = (1/2)∫(x dy - y dx) instead of a double integral. A participant struggles with integrating the expression for x derived from the curve and seeks advice on a more effective approach. It is suggested to find a suitable parameterization of the curve to simplify the integration process. The key takeaway is to focus on parameterizing the curve for easier application of the area formula.
Amy Marie
Messages
13
Reaction score
0

Homework Statement



Use, using the result that for a simple closed curve C in the plane the area enclosed is:

A = (1/2)∫(x dy - y dx) to find the area inside the curve x^(2/3) + y^(2/3) = 4

Homework Equations


Green's Theorem:
∫P dx + Q dy = ∫∫ dQ/dx - dP/dy

The Attempt at a Solution


I solved the equation of the curve for x:

x = (4 - y^(2/3))^(3/2)

Also, from the original curve equation x^(2/3) + y^(2/3) = 4, when x = 0, y = +/- 8 because 4^(3/2) = 8.

But when I plug x = +/- (4 - y^(2/3))^(3/2) in for the x bounds and y = +/- 8 in for the y bounds in the resulting double integral

(1/2)∫∫ 2 dxdy

I have trouble integrating x = (4 - y^(2/3))^(3/2) it with respect to y.

Does anybody happen to know if there is a more correct way to solve this problem?

Thank you for your help!
 
Physics news on Phys.org
The question does not want you to work the integral out by doing a double integral. It wants you to find a nice parameterization of the curve and do$$
A = \frac 1 2 \int_C x~dy - y~dx$$ So your first job is to find a nice parameterization. As a hint think about a way to parameterize it so that you can use the identity ##(r\cos\theta)^2 + (r\sin\theta)^2 = r^2##.
 
LCKurtz said:
The question does not want you to work the integral out by doing a double integral. It wants you to find a nice parameterization of the curve and do$$
A = \frac 1 2 \int_C x~dy - y~dx$$ So your first job is to find a nice parameterization. As a hint think about a way to parameterize it so that you can use the identity ##(r\cos\theta)^2 + (r\sin\theta)^2 = r^2##.
Thank you for your help!
 
Question: A clock's minute hand has length 4 and its hour hand has length 3. What is the distance between the tips at the moment when it is increasing most rapidly?(Putnam Exam Question) Answer: Making assumption that both the hands moves at constant angular velocities, the answer is ## \sqrt{7} .## But don't you think this assumption is somewhat doubtful and wrong?

Similar threads

Replies
3
Views
2K
  • · Replies 10 ·
Replies
10
Views
2K
Replies
12
Views
2K
Replies
4
Views
2K
  • · Replies 4 ·
Replies
4
Views
2K
Replies
15
Views
3K
Replies
3
Views
2K
Replies
5
Views
2K
  • · Replies 14 ·
Replies
14
Views
2K
  • · Replies 25 ·
Replies
25
Views
2K